Men & women Quiz

Descrição

Men & women Quiz (15 questions)
Mostapha Abdelkader
Quiz por Mostapha Abdelkader, atualizado more than 1 year ago
Mostapha Abdelkader
Criado por Mostapha Abdelkader quase 8 anos atrás
219
0

Resumo de Recurso

Questão 1

Questão
M.T. is a 72-year-old white man with a history of hypertension who admits smoking 1 pack of cigarettes per day. He states that he walks on his treadmill 30 minutes a day. He is 69 inches tall and weighs 150 lb (68 kg). His bone mineral density (BMD) T-score is –2.1 at the hip and –2.2 at the spine. His fracture history includes an adult fall at age 68 with an ankle fracture. His Fracture Risk Assessment Tool(FRAX) score (10-year fracture probability) is 14%,and his probability of hip fracture is 6.7%. Which best describes M.T.’s condition?
Responda
  • He has low bone mass (osteopenia) of the hip and spine.
  • He has osteoporosis of the spine and low bone mass (osteopenia) of the hip.
  • He has osteoporosis of the hip and spine.
  • He has normal BMD of the hip and spine.

Questão 2

Questão
M.T. is a 72-year-old white man with a history of hypertension who admits smoking 1 pack of cigarettes per day. He states that he walks on his treadmill 30 minutes a day. He is 69 inches tall and weighs 150 lb (68 kg). His bone mineral density (BMD) T-score is –2.1 at the hip and –2.2 at the spine. His fracture history includes an adult fall at age 68 with an ankle fracture. His Fracture Risk Assessment Tool(FRAX) score (10-year fracture probability) is 14%,and his probability of hip fracture is 6.7%. How to treat?
Responda
  • Take Ca1200 mg orally daily, vit. D 800 iu orally daily & alendronate 70 mg orally weekly.
  • Ca 1200 mg orally daily & vit. D 600 iu orally daily & begin weight-bearing exercise.
  • Ca 1200 mg orally daily, vit. D 600 iu orally daily, and raloxifene60 mg orally daily.
  • Ca 1200 mg orally daily, vit. D 400 iu orally daily, and risedronate 35 mg orally weekly & begin weight-bearing exercise.

Questão 3

Questão
A 29-year-old woman who is 65 inches tall and weighs 140 lb (63 kg) has a history of two deep venous thromboses (DVTs) but is otherwise healthy; she is seeking to become pregnant. She currently takes warfarin 3 mg orally daily. Which regimen is the best recommendation for this patient?
Responda
  • Continue current warfarin dose to prevent clots during pregnancy.
  • Continue warfarin therapy, but increase the dose to prevent clots during pregnancy.
  • Discontinue warfarin; start enoxaparin 40 mg SC daily until pregnant and continue through pregnancy.
  • Discontinue warfarin; start heparin 5000 units SC every 8 hours daily until 12 weeks pregnant, and then reinitiate warfarin.

Questão 4

Questão
J.K. is a 51-year-old postmenopausal woman suffering from severe hot flashes that have not resolved with venlafaxine 75 mg orally daily. She is otherwise healthy with no history of cancer and no surgical procedures. She is given conjugated estrogen 0.625 mg orally daily. Which statement is best for J.K.?
Responda
  • No other drug is required; estrogen alone is sufficient for hot flashes.
  • No other drug is required because J.K. is otherwise healthy & continue on venlafaxine.
  • Medroxyprogesterone acetate should be added to decrease the risk of stroke.
  • Medroxyprogesterone acetate should be added to decrease the risk of endometrial cancer.

Questão 5

Questão
Which of the following is true about DepoPovera (medroxyprogesterone acetate) ?
Responda
  • DepoProvera is given IV or SQ once a year
  • DepoProvera usually causes weight loss
  • DepoProvera can cause loss of bone
  • DepoProvera is contraindicated if breastfeeding

Questão 6

Questão
C.S. is a 49-year-old postmenopausal woman experiencing severe hot flashes, vaginal dryness, and pain during sexual intercourse. C.S. has a history of irregular uterine heavy bleeding, which resulted in a total hysterectomy 5 months ago. Her hot flshes are affecting her quality of life. Which treatment is best to recommend for C.S.?
Responda
  • Estradiol vaginal cream 0.1 mg/g.
  • Conjugated estrogen and medroxyprogesterone acetate (Prempro) 0.625 mg/2.5 mg tablets.
  • Conjugated estrogen (Premarin) 0.3-mg tablets.
  • Ospemifene (Osphena) 60-mg tablets.

Questão 7

Questão
S.F. is a 20-year-old woman initiated on ethinyl estradiol 30 mcg/drospirenone 3 mg oral tablets 5 months ago for contraception. She was recently prescribed lamotrigine for bipolar disorder. Which best describes the drug interaction that may occur with ethinyl estradiol/drospirenone and lamotrigine?
Responda
  • The effectiveness of ethinyl estradiol and drospirenone may be decreased.
  • The effectiveness of lamotrigine may be increased.
  • The effectiveness of lamotrigine may be decreased.
  • The effectiveness of ethinyl estradiol and drospirenone may be increased.

Questão 8

Questão
K.M. is a 28-year-old woman with a history of migraine with aura seeking contraception. She is 68 inches tall and weighs 215 lb (98 kg), with blood pressure (BP) today of 135/82 mm Hg; she denies smoking and alcohol use and states she would like to have children in a year or so. Which is the best contraceptive agent for K.M.?
Responda
  • Levonorgestrel intrauterine system (IUS).
  • Oral tablet norethindrone (Micronor).
  • Transdermal ethinyl estradiol/etonogestrel patch.
  • Oral ethinyl estradiol/desogestrel oral tablet (Mircette).

Questão 9

Questão
L.L. is a 38-year-old woman who has been trying to conceive for the past 7 months. Her husband’s medical examination is normal; L.L. is not ovulating every month. She has not tried any medications previously to induce ovulation. Which medication is best to initiate in L.L. to induce ovulation?
Responda
  • Ovidrel (human chorionic gonadotropin [hCG]).
  • Synarel (nafarelin–gonadotropin-releasing hormone [GnRH] agonist).
  • Pergonal (human menopausal gonadotropin [hMG]).
  • Clomid (Clomiphene).

Questão 10

Questão
T.G. is a 22-year-old woman who comes to a community pharmacy and requests emergency contraception (EC). She states that she was out of town for the weekend and was swimming when her contraceptive vaginal ring slipped out. She did not have a new one with her for replacement. She states she had unprotected intercourse 4 nights ago. She is worried about becoming pregnant. Which is the best recommendation for T.G.?
Responda
  • Recommend that she see her physician for a levonorgestrel 1.5 mg prescription.
  • Recommend EC; it may still be effective, she is within the 120-hour time window.
  • Do not recommend EC; it may be ineffective , she is beyond the 72-hour time window.
  • Do not recommend EC; instead, recommend that she insert a new vaginal ring.

Questão 11

Questão
The local health department requests your assistance with developing an osteoporosis treatment protocol for its interprofessional women's health clinic. The patient population served primarily includes independent living low-income elderly individuals. Databases provide a mechanism for identifying and monitoring patients through DXA (dual-energy absorptiometry) screenings and medication records When is the best time to reevaluate and consider discontinuation of bisphosphonate therapy for most postmenopausal women with osteoporosis?
Responda
  • indefinitely
  • 1 year
  • 3-5 years
  • 7 years

Questão 12

Questão
A 33-year-old woman with normogonadotropic normoestrogenic anovulation returns to the infertility specialist after seven cycles of clomiphene have failed. The patient has no pertinent medical history. Laboratory values from her last visit include blood glucose 97 mg/dL, thyroid-stimulating hormone 2.8 mcg/mL, and serum prolactin 12 mcg/L (upper limit of normal is 20 mcg/L). Which best represents an appropriate next step?
Responda
  • Gonadotropin
  • Metformin
  • Bromocriptine
  • Rosiglitazone

Questão 13

Questão
A 36-year-old woman (weight 82 kg, height 62 inches) presents to he physician seeking contraception. She is 3 weeks postpartum. Her child was delivered by(C-section). She is not planning to breastfeed. She does not want to conceive too soon after her newborn’s delivery. She wants to restart her previous prescription of ethinyl estradiol 0.030 mg and desogestrel 0.15 mg. Blood pressure today is 120/78 mm Hg. Her medical history includes depression. Current medications include oxycodone 5 mg/acetaminophen 325 mg orally every 4–6 hours as needed for pain and sertraline 50 mg orally daily. Which is the most appropriate time for the patient to restart ethinyl estradiol 0.030 mg and desogestrel 0.15 mg?
Responda
  • Now.
  • 4 weeks after delivery.
  • 6 weeks after delivery.
  • 12 weeks after delivery.

Questão 14

Questão
Patient pregnant her 2nd time she had abortion the first time and baby had neutrabifida....what is recommendations for her in next pregnancy?
Responda
  • (4000 micrograms) of folic acid daily beginning one month before they start trying to get pregnant and continuing through the first three months of pregnancy
  • (400 micrograms) of folic acid daily beginning one month before they start trying to get pregnant and continuing through the first three months of pregnancy
  • (2000 micrograms) of folic acid daily beginning one month before they starting to get pregnant and continue through the first three month of pregnancy
  • (200 micrograms) of folic acid daily beginning one month before they start trying to get pregnant and continuing through the first three months of pregnancy

Questão 15

Questão
30. T.M., a 33-year-old man, has a history of intravenous drug abuse and lives in and out of homeless shelters. He is taken to the emergency department by ambulance after experiencing paralysis on the right side of his body. The people at the shelter thought he might be having a stroke. In the emergency department, a laboratory profile was performed, which was positive for the Venereal Disease Research Laboratory test (syphilis test) with 10 white blood cells per cubic millimeter. T.M. has no known significant medical history (except for treatment of a sexually transmitted disease [STD]), but he is allergic to penicillin (anaphylactic reaction). Which therapy is best for T.M.?
Responda
  • Levofloxacin 750 mg intravenously × 1.
  • Penicillin G 4 million units every 4 hours intravenously for 14 days after penicillin desensitization.
  • Benzathine penicillin G 2.4 million units intramuscularly every week for 3 weeks after penicillin desensitization.
  • Azithromycin 500 mg intravenously or orally daily for 6 weeks.

Semelhante

The Tempest
Dirk Weibye
Discovery - HSC English
abby.slinger
Ecosystems at Risk
Lucy Chen
Translations and transformations of functions
Christine Laurich
HSC Chemistry Multiple Choice Quiz
megclark
Society's Perception of Divorced Women
Safeya Mohamed
Picasso
Corey Lance
History- Home Front WW1
jessmitchell
Waves
kate.siena
HSC Economics
lydia le
Computer science quiz
Ryan Barton